Math, asked by monjyotiboro, 11 hours ago

solve this;; mods/stars. ✴️✴️✨​

Attachments:

Answers

Answered by mathdude500
7

\large\underline{\sf{Solution-}}

➢ Given matrix is

\rm :\longmapsto\:A \:  =  \: \begin{gathered}\sf \left[\begin{array}{ccc}0&0& - 1\\0& - 1&0\\ - 1&0&0\end{array}\right]\end{gathered}

For part (a)

➢ To check whether inverse of matrix A exist or not, we have to check |A|.

So, Consider,

\rm :\longmapsto\: |A|

\rm \:  =  \:  \: 0 - 0 - 1(0 - 1)

\rm \:  =  \:  \: 1

Since,

\rm :\longmapsto\: |A| \:  \ne \: 0

Hence,

\rm :\longmapsto\: { |A| }^{ - 1}  \: exist.

For part (b) ,

We know,

➢ The value of identity matrix of order 3 × 3 is

\rm :\longmapsto\:I = \begin{gathered}\sf \left[\begin{array}{ccc}1&0&0\\0& 1&0\\ 0&0&1\end{array}\right]\end{gathered}

Now, given matrix is

\rm :\longmapsto\:A \:  =  \: \begin{gathered}\sf \left[\begin{array}{ccc}0&0& - 1\\0& - 1&0\\ - 1&0&0\end{array}\right]\end{gathered}

can be rewritten as

\rm :\longmapsto\:A \:  =  -  \: \begin{gathered}\sf \left[\begin{array}{ccc}0&0&1\\0&1&0\\  1&0&0\end{array}\right]\end{gathered}

\bf\implies \:A \:  \ne \:  -  \: I

For part (c)

We know,

➢ Unit matrix of order 3 × 3 is given by

\rm :\longmapsto\:I = \begin{gathered}\sf \left[\begin{array}{ccc}1&0&0\\0& 1&0\\ 0&0&1\end{array}\right]\end{gathered}

and

Given matrix is

\rm :\longmapsto\:A \:  =  \: \begin{gathered}\sf \left[\begin{array}{ccc}0&0& - 1\\0& - 1&0\\ - 1&0&0\end{array}\right]\end{gathered}

So,

\bf\implies \:A \:  \ne \: \: I

For part (d)

Given matrix is

\rm :\longmapsto\:A \:  =  \: \begin{gathered}\sf \left[\begin{array}{ccc}0&0& - 1\\0& - 1&0\\ - 1&0&0\end{array}\right]\end{gathered}

Now,

Consider,

\rm :\longmapsto\: {A}^{2}

\rm \:  =  \:  \: A \times A

\rm \:  =  \:  \: \begin{gathered}\sf \left[\begin{array}{ccc}0&0& - 1\\0& - 1&0\\ - 1&0&0\end{array}\right]\end{gathered} \times \begin{gathered}\sf \left[\begin{array}{ccc}0&0& - 1\\0& - 1&0\\ - 1&0&0\end{array}\right]\end{gathered}

\rm \:  =  \:  \: \begin{gathered}\sf \left[\begin{array}{ccc}0 + 0 + 1&0 + 0 + 0& 0 + 0 + 0\\0 + 0 + 0&0 + 1 + 0&0 + 0 + 0\\ 0 + 0 + 0&0 + 0 + 0&0 + 0 + 1\end{array}\right]\end{gathered}

\rm \:  =  \:  \: \begin{gathered}\sf \left[\begin{array}{ccc}1&0&0\\0& 1&0\\ 0&0&1\end{array}\right]\end{gathered}

\rm \:  =  \:  \: I

So,

\bf\implies \: {A}^{2} = I

Hence,

 \:  \:  \:  \:  \:  \:  \:  \:  \:  \:  \:  \:  \:  \:  \: \boxed{ \bf{ \: Option \: (d) \: is \: correct}}

Similar questions